IAS.NETWORK

PRELIMS 2021 TEST SERIES

TEST 4 ( 16th OCTOBER, 2020 )

© IAS.NETWORK

1. With reference to constitutional 3.Consider the following statements safeguards provided to the minorities regarding Epiphytes: under the Indian Constitution, consider the following statements: 1. They grow upon other plants merely for physical support. 1. Both religious and linguistic minorities 2. They are found in both tropical and have the Fundamental Right to establish temperate regions. educational institutions. 3. Mosses and ferns are common 2. The President should appoint a special epiphytes. officer for linguistic and religious minorities to investigate matters relating Which of the statements given above is/are to constitutional safeguards. correct? (a) 1 and 2 only Which of the statements given above (b) 1 and 3 only is/are correct? (c) 2 and 3 only (a) 1 only (d) 1, 2 and 3 (b) 2 only (c) Both 1 and 2 4. If Earth’s axis of rotation was 0 (d) Neither 1 nor 2 degree to the perpendicular to the plane of ecliptic instead of 23.5 degree, what 2. In India, the minerals are classified would have been the probable as minor minerals and major minerals. consequences? Which of the following is/are the prime basis for major minor classification? 1. There would have been no seasons on Earth. 1. End Use 2. The length of day and night would have 2. Availability of minerals been the same throughout the year. 3. The 3. Level of production length of day and night would have been 4. Export and Import the same all over the earth.

Select the correct answer using the code Select the correct answer using the code given below. given below. (a) 1 only (a) 1 and 2 only (b) 1, 3 and 4 only (b) 2 and 3 only (c) 2 and 3 only (c) 1 and 3 only (d) 1, 2, 3 and 4 (d) 1, 2 and 3

5. With reference to the decisions taken 7. Consider the following statements: during Tripuri session of Indian National Congress (INC), 1938, 1. While Central sector schemes are 100% consider the following statements: funded by the Union government, under Centrally Sponsored Scheme (CSS) a 1. Socialism was adopted as the official certain percentage of the funding is goal of INC. borne by the States. 2. The resolution on National Economic 2. While Central Sector Schemes are programme and Fundamental rights implemented by the Central Government were passed. machinery, Centrally Sponsored Scheme (CSS) is implemented by the State Which of the statements given above Governments. is/are correct? (a) 1 only Which of the statements given above is/are (b) 2 only correct? (c) Both 1 and 2 (a) 1 only (d) Neither 1 nor 2 (b) 2 only (c) Both 1 and 2 6. Consider the following statements: (d) Neither 1 nor 2

1. It is an indigenously developed nuclear capable missile. 8. Which of the following signifies the 2. It is a short range surface to surface Federal spirit of the Indian missile. Constitution? 3. It is the first missile to be developed by DRDO under Integrated Guided Missile 1. Article 345 Development Programme. 2. Article 371-B 3. Seventh Schedule The above properties refers to which of 4. Ninth Schedule the following missiles? (a) Prithvi II Select the correct answer using the code (b) Brahmos given below. (c) Agni (a) 1, 2, 3 only (d) Akash (b) 1, 3 and 4 only (c) 1, 4 and 3 only (d) 2, 3 and 4 only

11. Which of the following dynasties 9. Which of the following statements is is/are associated with Sanchi Stupa? not correct about the United Nations Convention to Combat Desertification? 1. Mauryan 2. Satavahanas (a) It is a not legally binding to its 3. Guptas signatories. (b) It encourages the participation of local Select the correct answer using the code people in combating desertification. given below. (c) It was established in accordance to the (a) 1 only recommendations of 1992 Rio Earth (b) 1 and 3 only Summit. (c) 2 and 3 only (d) India is a signatory to the convention. (d) 1, 2 and 3

10. Which of the following reasons can 12. Food grains stock in the Central Pool be attributed to the richness of Natural consists of: Vegetation in tropical rainforests: 1. Stock held by Food Corporation of India 1. Fertile Soil (FCI) 2. Intense sunlight for photosynthesis 2. States participating in the Decentralised 3. Wet climate Procurement Scheme 4. Seasonal changes to facilitate nutrient 3. State government agencies (SGAs) for absorption both buffer and operational requirements

Which of the statements given above Select the correct answer using the code is/are correct? given below. (a) 1 only (a) 1 and 4 only (b) 1 and 2 only (b) 2 and 3 only (c) 2 and 3 only (c) 1, 2 and 3 only (d) 1, 2 and 3 (d) 1, 2, 3 and 4

15. Which of the following can be 13. The recent Lion Census (2015) considered as the ultimate custodian of conducted in India recorded an the popular will of people in India? impressive rise in the number of lions. Which among the following methods (a) Parliament were adopted as part of this (b) Supreme Court enumeration? (c) Council of Ministers (d) Prime Minister 1. Physical view 2. Pugmarks 16. "Traditionally, in comparison to the 3. Voice records area sown during Rabi, farmers cultivate only about 50% of the area Select the correct answer using the code during the season of Kharif. The given below. primary reason given is that the region (a) 1 only suffers due to Anna Pratha." (b) 1 and 2 only (c) 2 and 3 only Which of the following regions in best (d) 1, 2 and 3 described by the above passage? (a) Marathwada 14. Which of the following leaders (b) Rayalaseema was/were not associated with the (c) Bundelkhand Non-cooperation movement? (d) Marusthali

1. Subhash Chandra Bose 17. Which of the following projects is/are 2. Bal Gangadhar Tilak related to providing last mile internet 3. Chittranjan Das connectivity?

Select the correct answer using the codes 1. Project Loon given below. 2. Aquila (a) 1 and 3 only 3. White Fi (b) 2 only (c) 2 and 3 only Select the correct answer using the code (d) 1, 2 and 3 given below. (a) 1 only (b) 1 and 3 only (c) 2 and 3 only (d) 1, 2 and 3

21. Tungabhadra doab was a resource 18. Which among the following rich region in southern India between correctly defines ‘Opportunity Cost’? the rivers Krishna and Tungabhadra. The region became a cause of conflict (a) The loss of other alternatives when between which of the following one alternative is not chosen. dynasties? (b) The economic cost of waiting for an opportunity. 1. Vijayanagara empire and Bahamani (c) A benefit of something that must be kingdom given up to acquire something else. 2. Yadavas and Hoysalas (d) The economic cost of operating a 3. Western Chalukyas and Cholas business. Select the correct answer using the code 19. In the 5th century BC, Buddha was given below. preaching in India. Who of the (a) 1 and 2 only following was not a contemporary of (b) 2 only Buddha? (c) 3 only (a) Purana Kassapa (d) 1, 2 and 3 (b) Rishabhdev (c) Ajita Kesakambalin 22. Which of the following rivers have (d) their origin in the Vindhyas?

20. "In the west it is bound by the 1. Betwa north-south spur of the Great 2. Parbati Himalayan Range which includes the 3. Ken world's third highest peak, 4. Banas Khangchendzonga and down to its south is Singalila ridge. In the north it Select the correct answer using the code is bound by Donghia range and also given below. partly includes the Tibetan Plateau. In (a) 1 and 3 only the east it is bound by the Chola (b) 1, 2 and 3 only range." (c) 1, 3 and 4 only (d) 2 and 4 only Which of the following states is best described by the above passage? (a) Arunachal Pradesh (b) Uttarakhand (c) Sikkim (d) Himachal Pradesh

23. Which of the following bills lapse 25. Which of the following principles is after dissolution of Lok Sabha? employed by a gymnast while performing a somersault? 1. A bill introduced in the Lok Sabha and (a) Conservation of Rotational Kinetic pending in Lok Sabha. Energy. 2. A bill passed by the Lok Sabha but (b) Conservation of Mass Moment of pending in the Rajya Sabha. Inertia. 3. A bill pending in the Rajya Sabha but (c) Conservation of angular momentum. not passed by the Lok Sabha. (d) Conservation of angular velocity.

Select the correct answer using the code given below. 26. Which of the following statements (a) 1 and 2 only are correct with regards to the (b) 1 and 3 only Compensatory Afforestation Fund Bill, (c) 2 and 3 only 2015? (d) 1, 2 and 3 1. It establishes National Compensatory 24. With reference to time and demand Fund under Public Account of India. deposits, which among the following 2. It aims to include an expert on tribal statements are correct? matters in both steering committee and executive committee of a State 1. All current account deposits are Authority. demand 3. It will be financed by companies that deposits. divert forest land. 2. All fixed deposits are time deposits. Select the correct answer from the code Select the correct answer using the code given below. given below. (a) 1 and 2 only (a) 1 only (b) 1 and 3 only (b) 2 only (c) 2 and 3 only (c) Both 1 and 2 (d) 1, 2 and 3 only (d) Neither 1 nor 2

29. With reference to Safety Valve 27. Travelling through which of the Theory of formation of Indian National following states are you likely to come Congress, consider the following across the Chimmony, Chinnar and statements: Chulanur Wildlife Sanctuaries? 1. It suggests that Congress was founded (a) Karnataka with the objective of saving the British (b) Tamil Nadu Empire in India. (c) Jammu and Kashmir 2. It was used by Lala Lajpat Rai to (d) Kerala criticise the Moderates.

28. Which of the following regions were Which of the statements given above is/are a part of the erstwhile Gondwanaland? correct? (a) 1 only 1. India (b) 2 only 2. Brazil (c) Both 1 and 2 3. South Africa (d) Neither 1 nor 2 4. Australia 30. With reference to the 'Forest Select the correct answer using the code Dwellers Act or Recognition of Forest given below. Rights Act, 2006', consider the following (a) 3 and 4 only statements: (b) 1, 2 and 3 only (c) 1 and 2 only 1. The implementing ministry of this act is (d) 1, 2, 3 and 4 Ministry of Tribal Affairs. 2. The Act seeks to recognize both the rights and occupation of traditional forest dwellers and Schedule Tribes in such forests. 3. It provides for diversion of forest land only with the recommendation of the gram sabhas.

Which of the statements given above is/are correct? (a) 1 only (b) 1 and 3 only (c) 2 and 3 only (d) 1, 2 and 3

31. Which among the following is the 34. Which of the following ancient texts most comprehensive measure of price deals with mathematics? rise? 1. Lilavati (a) Wholesale Price Index 2. Siddhanta Shiromani (b) Producer Price Index 3. Sulvasutra (c) GDP Deflator 4. Charak Samhita (d) Consumer Price Index Select the correct answer using the code 32. With reference to Concentrating given below. Solar Power technology, consider the (a) 1, 2 and 3 only following statements: (b) 1 and 2 only (c) 3 and 4 only 1. It does not use photovoltaic cells to (d) 1, 2, 3 and 4 generate power. 2. In this heat energy is converted to 35. Which among the following have a electricity. role in notifying an area as a tiger 3. It produces electricity in the form of reserve? alternating current. 1. National Tiger Conservation Authority Which of the statements given above 2. State government is/are 3. National Board for Wildlife correct? (a) 1 and 2 only Select the correct answer using the code (b) 1 and 3 only given below. (c) 2 and 3 only (a) 1 only (d) 1, 2 and 3 (b) 1 and 2 only (c) 1 and 3 only 33. Which of the following best (d) 1, 2 and 3 describes 'Barchans'? 36. Which among the following deficits (a) thick deposits of glacial clay indicate how much government (b) crescent-shaped sand dune borrowing is going to meet expenses (c) calcareous deposits in soil other than interest payments? (d) summer grasslands in the higher (a) Budget Deficit reaches (b) Fiscal Deficit (c) Primary Deficit (d) Revenue Deficit

37. Which of the following correctly defines 'Masala bonds'? 39. Which of the following is/are part of our State emblem? (a) They are rupee-denominated instruments sold only to offshore 1. Three visible lions investors by Indian corporates to raise 2. A galloping horse money from overseas. 3. A bull (b) They are dollar-denominated 4. A Chakra (Wheel) instruments sold only to offshore 5. A bell Shaped Lotus investors by Indian corporates to raise 6. Ahimsa Paramo Dharma written in money from overseas. Devanagari (c) They are rupee-denominated instruments sold only to Indian investors Select the correct option using the code by offshore corporates to raise money given below. from India. (a) 1, 2, 3 and 4 only (d) They are dollar-denominated (b) 5 and 6 only instruments sold only to Indian investors (c) 1, 2, 3, 4 and 5 only by offshore corporates to raise money (d) 1, 2, 3, 4, 5 and 6 from India. 40. With reference to 'Sidereal time' 38. Which of the following has been consider the following statements: adopted from the Westminster system of government in Indian Constitution? 1. It is based on the Earth's rate of rotation measured relative to the fixed stars 1. Parliamentary form of Government rather than the Sun. 2. Fundamental Rights 2. A sidereal day is shorter than the solar 3. Federal Structure day. 4. Judicial Independence Which of the statements given above is/are Select the correct answer using the code correct? given below. (a) 1 only (a) 1 only (b) 2 only (b) 2, 3 and 4 only (c) Both 1 and 2 (c) 1 and 2 only (d) Neither 1 nor 2 (d) 1 and 4 only

41. Which of the following statements related to "dioxins" is/ are true? 1. They are a type of persistent organic 44. With reference to election pollutants. methods/systems used in election of 2. They are a carcinogenic compound. President of India, consider the 3. Humans exposure is mainly through following: meat and dairy products. 1. Single Transferable vote Select the correct answer using the code 2. Proportional Representation given below. 3. First Past the Post System (a) 1 and 2 only 4. Voting by secret ballot (b) 1 and 3only (c) 2 and 3 only Which of the statements given above is/are (d) 1, 2 and 3 correct? (a) 1, 2 and 3 only 42. In some pockets of United Province, (b) 2, 3 and 4 only The peasants' movement was (c) 1, 2 and 4 only characterized by the Nai-Dhobi Band. (d) 1, 2, 3 and 4 Which of the following statements explains the Nai-Dhobi band? 45. With reference to United Nations (a) It was a form of political boycott. Environment Programme, consider the (b) It was a form of social boycott. following statements: (c) It was a form of economic boycott. (d) None of the above 1. It assesses global environmental conditions and trends. 43. Which of the following statements 2. It releases the Global Environment related to the chemical Muriate of Outlook report. Potash (Potassium Chloride) is/are correct? Which of the statements given above is/are 1. It is used as a crop fertilizer. correct? 2. It is used in rocket propellants and gun (a) 1 only powder. (b) 2 only (c) Both 1 and 2 Select the correct answer using the code (d) Neither 1 nor 2 given below. (a) 1 only (b) 2 only (c) Both 1 and 2 (d) Neither 1 nor 2

46. A sudden fall in Barometer reading 49. With reference to medieval India, indicates which of the following what were ‘manigram’ and ‘nanadesi’? regions? (a) Villages for revenue collection (b) Guilds of merchants (a) Stormy Weather (c) Village headmen (b) Calm Weather (d) Religious priests (c) Cold and Dry Weather (d) Hot and Sunny Weather 50. Xenotransplantation involves the transplantation of which of the following parts between different species? 47. Which of the following statements 1. Organs regarding Infrastructure Investment 2. Cells Trust (InvITs) is not correct? 3. Tissues

(a) These are mutual fund like institutions Select the correct answer using the code that enable investments into the given below. infrastructure sector. (a) 1 and 3 only (b) They are regulated by the Reserve (b) 1 and 2 only Bank of India. (c) 2 only (c) Listing on stock exchange is (d) 1, 2 and 3 mandatory for both publicly offered and privately placed InvITs. 51. Democratically elected governments (d) It will enable the investors to hold a are essentially the will of the majority. diversified portfolio of infrastructure In this context, which of the following assets. prevents the tyranny of majority in India? 48. Recently launched "Project Sunrise" by Union Government is 1. Rule of law related to: 2. Fundamental rights (a) National Solar Mission of Government 3. Separation of power of India. 4. Directive Principles (b) Eradication of Tuberculosis from India. Select the correct answer using the code (c) Prevention of AIDS in North Eastern given below. states. (a) 1 only (d) Development of Tourism industry in (b) 2, 3 and 4 only India. (c) 1, 2 and 3 only (d) 1, 2, 3 and 4

52. "It is a colourful bird, glossy pitch 54. With reference to Fiat Money, which black with yellow legs, orange and of the following statements are correct? yellow beak, bright yellow wattles on the head and a dash of white on the side 1. It is the currency that a government has wings. It is seen in pairs or noisy flocks declared to be legal tender. in preferred locations in forests and 2. It risks becoming worthless due to villages. Fondly called the mimic bird, hyperinflation. it talks, screeches, wails and whistles 3. It is convertible and can be redeemed. like a human. As a State bird of 4. Most modern currencies are fiat Chattisgarh it was on the brink of currencies. extinction due its preference as a favourite pet by the majority of the Select the correct answer using the code people." given below. (a) 1, 2 and 3 only Which of the following birds is best (b) 1, 2 and 4 only described by the above passage? (c) 3 and 4 only (a) Hill Mynah (d) 1, 2, 3 and 4 (b) Black Necked Crane (c) Himalayan Quail 55. Consider the following statements: (d) Indian Parakeet 1. While Type 1 diabetes affects children, 53. With reference to the salinity of the Type 2 is usually discovered in adulthood. surface waters of the ocean, consider 2. While Type 1 diabetes is an autoimmune the following statements: disease, Type 2 is believed to be a lifestyle disease. 1. Salinity in equatorial areas is lower 3. Type 1 diabetes is more prevalent in than in the tropical areas. India than Type 2. 2. Salinity in polar areas is lower than in the temperate areas. Which of the statements given above is/are correct? Which of the statements given above (a) 1 and 2 only is/are correct? (b) 1 and 3 only (a) 1 only (c) 2 and 3 only (b) 2 only (d) 1, 2 and 3 (c) Both 1 and 2 (d) Neither 1 nor 2

58. With reference to the Central 56. Sardar Vallabhbhai Patel was Vigilance Commission, consider the associated with which of the following? following statements:

1. Kheda Satyagraha 1. It is the main agency for preventing 2. Bardoli Satyagraha corruption in the Central government. 3. Nagpur flag Satyagraha 2. Appointment and removal of its members are done by the President. Select the correct answer using the code 3. It has to present annually to the Central given below. government a report on its performance. (a) 1 and 2 only (b) 1 and 3 only Which of the statements given above is/are (c) 2 and 3 only correct? (d) 1, 2 and 3 (a) 1 only (b) 1 and 2 only (c) 2 and 3 only (d) 1, 2 and 3 57. Which of the following best defines the gradual change in certain characteristics exhibited by 59. With reference to Sangam literature, communities along with gradual consider the following statements: change in one or more environmental gradients? 1. It is a religious literature. 2. It consist of poems composed in praise (a) Ecotone of warriors. (b) Ecotype 3. It provides information regarding long (c) Ecocline distance trade during the period. (d) Ecological Niche Which of the statements given above is/are correct? (a) 1 and 3 only (b) 2 only (c) 2 and 3 only (d) 1, 2 and 3

60. Which of the following is the objective of Advance Pricing 63. With reference to forest fires in Agreement (APA) programme? India, consider the following statements:

(a) To dictate the price of a good or 1. In the last five years, Uttarakhand has service in advance by governing agency registered the maximum incidents of so as to check price rise in essential goods forest fires. and services. 2. Forest fires in India usually peak during (b) Pre-determine the distribution of the dry months of December and natural resources between the government January. and a resource extraction company. 3. As a natural process, they help forests by (c) Pre-determine the distribution of promoting flowering and seedling profits and losses between a government establishment. and a resource extraction company. (d) To provide a predictable and non Which of the statements given above is/are adversarial tax regime and to reduce correct? litigation in the Indian transfer pricing (a) 1 only Regime. (b) 1 and 2 only (c) 3 only 61. Milankovitch Cycles are related to (d) 1, 2 and 3 which of the following? (a) Sunspot cycle (b) Variations in the Earth's eccentricity, 64. Economic Council for Asia and axial tilt, and precession Pacific works under the framework of: (c) Period of continental drift (a) United Nations (d) Geo-magnetic reversal cycle of Earth (b) Asia Pacific Economic Forum (c) South Asian Association for Regional 62. In a vacuum container a coin and a Cooperation leaf are dropped simultaneously. (d) World Bank Which of the following is most likely to happen?

(a) Coin will reach first at the bottom. (b) Leaf will reach first at the bottom. (c) Both will reach the bottom simultaneously. (d) Both will remain suspended in the container

65. Consider the following statements: 67. Which of the following is/are examples of Piedmont Plateau? 1. Like the Speaker, the Deputy Speaker is elected by the assembly itself from 1. Patagonian plateau amongst its members. 2. Deccan plateau 2. Like the Speaker, the Deputy Speaker 3. Tibetan plateau remains in office usually during the life of the assembly. Select the correct answer using the code given below. Which of the statements given above (a) 1 only is/are correct? (b) 2 and 3 only (a) 1 only (c) 1 and 3 only (b) 2 only (d) 1, 2 and 3 (c) Both 1 and 2 (d) Neither 1 nor 2 68. With reference to Pabna Agrarian unrest of 1870s, consider the following statements: 66. Which of the following describes 'Skewflation'? 1. It emerged against zamindars efforts to (a) across-the-board price increase over a prevent tenant from acquiring occupancy sustained period of time. rights. (b) an episodic price rise pertaining to one 2. It used legal resistance as main form of or a small group of commodities. struggle. (c) a price rise of one or a small group of 3. No anti-British demands were raised commodities over a sustained period of during the unrest. time. (d) across-the-board price decrease over a Which of the statements given above is/are sustained period of time. correct? (a) 1 and 2 only (b) 1 and 3 only (c) 2 and 3 only (d) 1, 2 and 3

69. Which of the following pairs are 71. With reference to Radio Frequency correctly matched? Identification (RFID), consider the following statements: 1. Patua art : Odisha 2. Pattachitra : West Bengal 1. It required direct line of sight between 3. Thangka : Sikkim the RFID tag and reader. 4. Kohvar : Jharkhand 2. Information on the tag cannot be altered. 3. RFID reader can read even up to 100 Select the correct answer using the code feet. given below. (a) 1 and 2 only Which of the statements given above is/are (b) 2 and 3 only correct? (c) 3 and 4 only (a) 1 and 2 only (d) 1, 3 and 4 only (b) 2 and 3 only (c) 3 only (d) 1, 2 and 3

70. Which of the following marine 72. A citizen of the United Kingdom is national park/wildlife sanctuary is not staying in India with his family. During located off the coast of mainland India? his stay he faces following issues:

(a) Gulf of Mannar Marine National Park 1. He is denied to work in a PSU on the (b) Gahirmatha Marine Wildlife grounds of him being not from India. Sanctuary 2. Being a Catholic, he wants to open a (c) Mahatma Gandhi Marine National Catholic School but is denied the Park permission. (d) Malvan Marine Wildlife Sanctuary 3. His wife writes a blog criticizing other religions and is arrested for it.

For which of the above he can directly go to the Supreme Court as a matter of violation of Fundamental Rights? (a) 1 only (b) 2 and 3 only (c) 3 only (d) None

75. With reference to the 'Integrated 73. Which of the following does not Gasification Combined Cycle', consider constitute a collective investment the following statements: scheme in India? 1. It uses Syngas to fuel combustion 1. Mutual Funds turbine. 2. Chit Funds 2. It reuses the exhaust heat from the 3. Deposits accepted by non-banking combustion turbine. financial companies Which of the statements given above is / Select the correct answer using the code are correct? given below. (a) 1 only (a) 1 only (b) 2 only (b) 1 and 3 only (c) Both 1 and 2 (c) 2 and 3 only (d) Neither 1 nor 2 (d) 1, 2 and 3 76. India’s first indigenous rotavirus vaccine named Rotavac is launched to 74. If one travels from West to east, combat infant mortality caused due to which of the following will be the which of the following diseases? correct sequence of hills/range that (a) Pneumonia he/she comes across? (b) Diarrhea (c) Measles (a) Mizo Hills, Barail Range, Naga Hills, (d) Tuberculosis Mishmi Hills (b) Barail Range, Mizo Hills, Naga Hills, 77. The followers of this school of Mishmi Hills philosophy are described as fatalists as (c) Mizo Hills, Barail Range, Mishmi they believed that everything is Hills, predetermined in this world and human Naga Hills beings have no free will. The exact (d) Mishmi Hills, Naga Hills, Barail nature of doctrine of this philosophy is Range, not very clear because the sect's own Mizo Hills texts have not survived. Which is the school of philosophy? (a) Lokayata (b) Ajivika (c) Advaita Vedanta (d) Ahetuvadin

78. Which of the following statements 80. Which of the following is correct related to the "LVM 3" launch vehicle regarding the recently launched 'Setu being developed by Indian Space Bharatam Project'? Research Organisation is/are correct? (a) It aims to make all national highways 1. It uses an indigenously developed free of railway crossings. cryogenic engine (b) It aims to ensure railway connectivity 2. It is capable of placing with all the major ports. Geosynchronous satellites into orbit. (c) It aims to create a shipping route in the shallow straits between India and Sri Select the correct answer using the code Lanka. given below. (d) It aims to connect all the National (a) 1 only waterways either through railways or (b) 2 only roadways. (c) Both 1 and 2 (d) Neither 1 nor 2 81. Consider the following statements regarding President’s Rule: 79. In Cricket, fast bowlers often shine only one side of the cricket bowl to 1. It is proclaimed by the President only on generate swing. the report of the Governor. 2. Its revocation requires simple majority Which of the following could be the of both the Houses of Parliament. reason? 3. The laws made during President's Rule (a) Air travels faster on shiny side and put continue to be in operation even after the lower pressure compared to rough side. President’s rule. (b) Air travels faster on the rough side and puts lower pressure compared to the Select the correct answer using the code shiny side. given below. (c) Air travels faster on the rough side and (a) 1 and 2 only puts higher pressure compared to the (b) 2 and 3 only shiny side. (c) 3 only (d) Air travels faster on shiny side and put (d) 1, 2 and 3 higher pressure compared to the rough side.

82. Consider the following statements 84. Which of the following parameters regarding Central Road Fund: specified by the Central Pollution Control Board are typically used for 1. It is a statutory non-lapsable fund. specifying drinking water quality in 2. It is created out of cess imposed on the India? consumption of Petrol and High Speed Diesel. 1. Total Coliform content 3. It is levied to develop and maintain 2. pH value National Highways and railway 3. Dissolved Oxygen under/over bridges. 4. Biochemical Oxygen Demand

Which of the statements given above Select the correct answer using the code is/are correct? given below. (a) 1 and 3 only (a) 1, 3 and 4 only (b) 2 only (b) 1 and 4 only (c) 3 only (c) 2 and 3 only (d) 1, 2 and 3 (d) 1, 2, 3 and 4

83. Which of the following countries 85. Consider the following statements are not members of the regarding LED TVs. Non-Proliferation Treaty (NPT)? 1. They utilize liquid crystal display 1. India technology. 2. Pakistan 2. The screen is lit by fluorescent tubes. 3. Israel 4. China Which of the statements given above is / are correct? Select the correct answer using the code (a) 1 only given below. (b) 2 only (a) 1 and 2 only (c) Both 1 and 2 (b) 1, 2 and 3 only (d) Neither 1 nor 2 (c) 1 and 3 only (d) 2, 3 and 4 only

86. With reference to Indian Social 88. Which of the following Committees Conference, consider the following is/are formed only in Lok Sabha? statements: 1. Committee of Privileges 1. It was founded by M.G. Ranade and 2. Committee on Absence of members Raghunath Rao. 3. Committee on Private Members' Bills 2. It aimed to bring together all the social and Resolutions reform groups of the country under one organization. Select the correct answer using the code 3. It started Pledge movement against given below. child marriage. (a) 1 only (b) 1 and 2 only Which of the statements given above (c) 2 and 3 only is/are correct? (d) 1, 2 and 3 (a) 1 and 2 only (b) 1 and 3 only (c) 2 and 3 only 89. Which of the following best describes (d) 1, 2 and 3 Open Acreage Licensing Policy (OALP)? 87. Africa and Eurasia touches at which of the following points/ (a) It is a hydrocarbon exploration policy interface? which provides a company an opportunity to select the exploration blocks, without 1. Strait of Gibraltar waiting for the formal bid round from the 2. Suez Canal Government. 3. Straits of Hormuz (b) It is a telecom spectrum distribution 4. Bab-El-Mandeb policy where government invites even non-telecom companies to bid for unused Select the correct answer using the code spectrums known as White-Fi. given below. (c) It is a defence procurement policy in (a) 1 and 2 only which government invites foreign (b) 1, 2 and 4 only companies to invest in India with mandated (c) 2 and 4 only offset requirements. (d) 3 and 4 only (d) It is a public procurement policy where government fixes the revenue share model in a contract farming agreement to check the exploitation of farmers by private companies

90. Consider the following pairs: 92. 'Sick Building Syndrome' is Tropical Grasslands : Regions associated with which of the following?

1. Llanos : Venezuela (a) disaster-vulnerability due to 2. Pampas : Brazil noncompliance with building codes 3. Campos : Argentina (b) health problems due to indoor air pollution Which of the above pairs are correctly (c) non-compliance with Floor-Area Ratio matched? stipulations (a) 1 only (d) buildings having greater carbon (b) 1 and 2 only footprint (c) 1 and 3 only (d) 2 and 3 only 93. Which among the following are tributaries of Yamuna? 91. With reference to Kalighat paintings, consider the following 1. Tons statements: 2. Ken 3. Betwa 1. It originated in West Bengal. 4. Son 2. It depicts figures in three-dimensional form. Select the correct answer using the code 3. Social changes introduced by British given below. were used as its theme. (a) 1, 2 and 3 only (b) 2, 3 and 4 only Which of the statements given above (c) 1 and 4 only is/are correct? (d) 1, 2, 3 and 4 (a) 1 only (b) 1 and 3 only (c) 2 and 3 only (d) 1, 2 and 3

94. When we travel by bus or train, we 96. The government, recently has given notice that objects which are nearer to its nod to the ‘Comprehensive us move faster than the objects which Integrated Border Management System‘ are far. Which of the following explains (CIBMS) for 24x7x365 surveillance the above observation? through technology for which of the following borders? (a) Relative velocity decreases with (a) China increase in distance. (b) Pakistan (b) Relative angular velocity decreases (c) Bangladesh with increase in distance. (d) Myanmar (c) Distant objects are difficult to observe by eyes compared to the nearer objects. (d) None 97. With reference to modern India, Treaty of Sugauli was: (a) a treaty to end the first Anglo-Maratha 95. With reference to Overseas citizen war signed between British and Marathas. of India (OCI), consider the following (b) a treaty for establishing boundary of statements: Nepal signed between British and Nepal. (c) a treaty to end Second Anglo Mysore 1. A person is registered as an OCI under war signed between British and Tipu Citizenship Act, 1955. Sultan. 2. An OCI has to register with local police (d) a treaty to allow British to collect taxes if his stay in India exceeds 180 days. in Bengal, Bihar and Orissa. 3. OCI enjoy parity with non-resident Indians in all matters. 98. Economic growth is usually coupled with inflation. Which of the following Which of the statements given above factors may be responsible for this? is/are correct? (a) 1 only 1. Increased Aggregate Demand (b) 1 and 2 only 2. Increasing wages (c) 2 and 3 only 3. Higher employment (d) 1, 2 and 3 Select the correct answer using the code given below. (a) 1 and 2 only (b) 2 and 3 only (c) 1 and 3 only (d) 1, 2 and 3

99. Among the following organisms, which one does not belong to the Phylum of other three?

(a) Bed bug (b) Bee (c) Bat (d) Butterfly

100. The tribal communities Meiteis, Nagas and Kukis are found in which of the following states?

(a) Tripura (b) Manipur (c) Arunachal Pradesh (d) Assam

ANSWER KEY AND EXPLANATION

1. (A) Article 30 grants the following rights to minorities, whether religious or linguistic: All minorities shall have the right to establish and administer educational institutions of their choice. The compensation amount fixed by the State for the compulsory acquisition of any property of a minority educational institutions shall not restrict or abrogate the right guaranteed to them. This provision was added by the 44th Amendment Act of 1978 to protect the right of minorities in this regard. The Act deleted the right to property as a Fundamental Right (Article 31). In granting aid, the State shall not discriminate against any educational institution managed by a minority. Thus, the protection under Article 30 is confined only to minorities (religious or linguistic) and does not extend to any section of citizens (as under Article 29). The Seventh Constitutional Amendment Act of 1956 inserted a new Article 350-B in Part XVII of the Constitution. This article contains the following provisions: There should be a Special Officer for Linguistic Minorities. He is to be appointed by the President of India. It would be the duty of the Special Officer to investigate all matters relating to the safeguards provided for linguistic minorities under the Constitution. Thus, the provision of appointing Special Officer pertains only to linguistic minorities and not religious minorities,

2. (A) In India, the minerals are classified as minor minerals and major minerals. The major-minor classification has nothing to do with the quantum /availability of these minerals, though it is correlated with the relative value of these minerals. Further, this classification is based more on their end use, rather than level of production, level of mechanization, export and import etc. (eg. Sand can be a major mineral or a minor mineral depending on where it is used; same is the case for limestone.) The power to frame policy and legislation relating to minor minerals is entirely delegated to the State Governments while policy and legislation relating to the major minerals are dealt by the Ministry of Mines under Union /Central Government.

3. (D) Epiphytes are plants that grow upon another plant or object merely for physical support. They are not parasitic on the supporting plants and they produce their own energy from photosynthesis and obtain moisture and nutrients from the air. They do not damage a host directly, although their attachment roots and bulk can eventually ‘strangle’ or topple large trees. Mosses, ferns, and liverworts are also common epiphytes and are found in both tropical and temperate regions. Therefore, all statements are correct

4. (D) Effect of the inclined axis on the duration of day and night: The duration of day and night is not equal at all places. The time of the day at any place is determined by the height of the sun in the sky. If the axis of the earth was vertical instead of inclined, places on earth would have the same temperature throughout the year. If the earth did not revolve around the sun even then with its inclined axis there would be no seasons.

Effect of the inclination of the axis of seasons: The path of the earth around the sun is known as elliptic. It passes through the centre of both the earth and the sun. The earth’s axis is not perpendicular to the plane of the elliptic, but it is tilted at an angle is 66½0 to it. The axis is, therefore, inclined at an angle of 23½0 from the perpendicular to this plane. As a result of this, in the course of the earth’s movement around the sun, the North pole remains inclined towards the sun for a part of the year while the South Pole is titled away from it. For the remaining part of the year, the situation is reversed, the North Pole being tilted away from the sun and the South Pole being inclined towards it. On earth this gives rise to the changing seasons and the varying lengths of day and night. If the earth’s axis were perpendicular to the plane of orbit, there would be equal nights and days at all places on the earth, at all times of the year and there would also be no seasons.

5. (D) The presidential election to the Tripuri Congress session took place early in 1939. Subash Chandra Bose was re - elected defeating Dr Pattabhi Sitaramayaa who had been backed by Mahatma Gandhi. He resigned in April 1939, and announced in May 1939 the formation of the Forward Bloc within the Congress. Socialism did not at any stage become the official goal of the Indian National Congress though there was a great deal of debate around it within the national movement and the Indian National Congress during the 1930s and 1940s. National Economic programme and resolution on Fundamental rights were passed at karachi session, 1931.

6. (A) Prithvi II is an indigenously developed nuclear capable missile. With a strike range of 350 km, the Prithvi-II is capable of carrying 500 kg to 1,000 kg of warheads and is thrusted by liquid propulsion twin engines. It uses an advanced inertial guidance system with manoeuvring trajectory to hit its target. It is a short range surface to surface missile. Inducted into Indian armed forces in 2003, the nine-metre-tall, single-stage liquid-fuelled Prithvi-II is the first missile to be developed by the DRDO under the Integrated Guided Missile Development Programme.

7. (C) In India’s development plan exercise we have two types of schemes viz; central sector and centrally sponsored scheme. The nomenclature is derived from the pattern of funding and the modality for implementation. Under Central sector schemes, it is 100% funded by the Union government and implemented by the Central Government machinery. Under Centrally Sponsored Scheme (CSS) a certain percentage of the funding is borne by the States in the ratio of 50:50, 70:30, 75:25 or 90:10 and the implementation is by the State Governments. Therefore, both statements are correct.

8. (A) Art. 345: The Legislature of a State may by law adopt any one or more of the languages in use in the State or Hindi as the language or languages to be used for all or any of the official purposes of that State: Provided that, until the Legislature of the State otherwise provides by law, the English language shall continue to be used for those official purposes within the State for which it was being used immediately before the commencement of this Constitution. Art. 371B: It is a special provision with respect to the State of Assam. It is mostly related to the functioning of the Legislative Assembly and Sixth schedule. Under this the President may provide for the constitution and functions of a committee of the Legislative Assembly of the State consisting of members of that Assembly elected from the tribal areas of the Sixth Schedule and such number of other members of that Assembly as may be specified in the order and for the modifications to be made in the rules of procedure of that Assembly for the constitution and proper functioning of such committee. In a federal polity subjects/powers are divided between Central government and States as enshrined under the Seventh Schedule to the Indian Constitution. Thus, all the above signifies federal spirit of the Indian Constitution, except the Ninth Schedule, which provides that articles/laws mentioned in this schedule are immune from Judicial Review.

9. (A) Desertification, along with climate change and the loss of biodiversity, were identified as the greatest challenges to sustainable development during the 1992 Rio Earth Summit. Established in 1994, UNCCD is the sole legally binding international agreement linking environment and development to sustainable land management. The Convention addresses specifically the arid, semiarid and dry sub-humid areas, known as the drylands, where some of the most vulnerable ecosystems and peoples can be found. The Convention’s 195 parties work together to improve the living conditions for people in drylands, to maintain and restore land and soil productivity, and to mitigate the effects of drought. The UNCCD is particularly committed to a bottom-up approach, encouraging the participation of local people in combating desertification and land degradation. The UNCCD secretariat facilitates cooperation between developed and developing countries, particularly around knowledge and technology transfer for sustainable land management. As the dynamics of land, climate and biodiversity are intimately connected, the UNCCD collaborates closely with the other two Rio Conventions; the Convention on Biological Diversity (CBD) and the United Nations Framework Convention on Climate Change (UNFCCC), to meet these complex challenges with an integrated approach and the best possible use of natural resources.

10. (B) The soil of tropical rainforests is not fertile. It's one of the reasons if a tropical rainforest is removed, it does not regenerate quickly as compared to a tropical deciduous forest. Intense sunlight for photosynthesis and Wet climate helps in Natural Vegetation in tropical rainforests. Equatorial zones have very low seasonal variability as it remains the same throughout the year. Hence B is the correct answer.

11. (D) It was Emperor Asoka who laid the foundations of a religious centre at Sanchi. During Sunga times, several edifices were raised at Sanchi and its surrounding hills. The Asokan Stupa was enlarged. In the first century BC the Andhra-Satavahanas, who had extended their sway over the eastern Malwa, caused the elaborately carved gateways to Stupa 1. From the second to fourth century AD Sanchi and Vidisha came under the Kushanas and Kshatrapas and subsequently passed on to the hands of the Guptas. During the Gupta period some temples were also built and sculptures were added displaying the classical grace and simplicity of the era. Thus Sanchi displays harmonious co-existence of Hindu and Buddhist faiths.

12. (D) Food grains stock in the Central Pool consists of stock held by Food Corporation of India (FCI), states participating in the Decentralised Procurement Scheme and the state government agencies (SGAs) for both buffer and operational requirements. The total annual stock of foodgrains in the Central Pool is distributed over different quarters of the year depending upon off-take and procurement patterns. The seasonality of production and procurement is thus, a decisive factor in determining the minimum norm of food grains stocks required in a particular quarter of the year.

13. (A) In order to avoid the danger of overestimation, new methods were adopted in the recent (14th) lion census. The enumerators were GPS enabled and only recorded lions when they saw them physically and not through pugmarks or any other signs like hearing a roar.

14. (B) Statement 1 is correct. The NCM was a great success in Bengal, especially in Calcutta. The educational boycott was particularly very successful. Subhas Bose became the principal of the National Congress in Calcutta. Bal Gangadhar Tilak died long before the idea of NCM was conceived by Congress. On 4 September 1920, Congress met at Calcutta in a special session. This special session was presided over by Lala Lajpat Rai. In this session Gandhi projected that if the Non-cooperation movement gets successful, Swaraj could be attained in One year. This was something immediately repudiated by CR Das. In December 1920, Congress met once again in the Nagpur Session. This time the differences of CR das had melted away. He moved the main resolution of Non-cooperation. CR Das was an important personality in Bengal during the time of the non-cooperation movement from 1919 to 1922

15. (A) Of the three organs of the government, executive is generally associated with the reason of the state, judiciary with the guardianship of the constitution and the parliament is the ultimate custodian of the popular will.

16. (C) Traditionally, in comparison to the area sown during Rabi, farmers cultivate only about 50% of the area during the season of Kharif in Bundelkhand. While the net cropped area during Rabi is approximately 18.50 lakh hectares, the area cropped during Kharif has hovered around 9 lakh hectares. The primary reason given is that the region suffers due to Anna Pratha. This is a traditional system under which people leave their milch cattle unfettered to graze in the fields. These cattle eat up the crops and therefore the farmers are not inclined to sow their lands during the season of Kharif. This does not happen during the Rabi season as the farmers keep their cattle at home.

17. (D) Project Loon is a research and development project being developed by X (formerly Google X) with the mission of providing Internet access to rural and remote areas. Aquila by Facebook is a solar power plane that beams internet connectivity from the Sky. White Fi by Microsoft uses unused spectrum to provide connectivity.

18. (C) An opportunity cost is the cost of an alternative that must be forgone in order to pursue a certain action. Put another way, the benefits you could have received by taking an alternative action. Since every resource can be put to alternative uses, every action or decision has an associated opportunity cost. The opportunity cost of going to college is the money you would have earned if you worked instead. On the one hand, you lose four years of salary while getting your degree; on the other hand, you hope to earn more during your career, thanks to your education, to offset the lost wages. Another example: if a gardener decides to grow carrots, his or her opportunity cost is the alternative crop that might have been grown instead (potatoes, tomatoes, pumpkins, etc.)

19. (B) Rishabhdev (Rishabhanatha) was the first Tirthankara of Jains. Mahavir the 24th Tirthankara was contemporary of Buddha. Purana Kassapa was an Indian ascetic teacher who lived around the 5th or 4th century BCE, contemporaneous with and the Buddha. Purana taught a theory of "non-action" whereby the body acts independent of the soul, or demerit. In the Canon, Purana ( is identified as an ahetuvadin, "denier of a cause". was an ancient Indian philosopher, a contemporary of Buddha and Mahavira. It has frequently been noted that the doctrines of the Lokayata school were considerably drawn from Ajita's teachings. Makkhali Gosala or Manthaliputra Goshalak was an ascetic teacher of ancient India. He was a contemporary of Buddha and of Mahavira. He is considered as the founder of Ajivika philosophy.

20. (C) Sikkim, in the west is bound by the north-south spur of the Great Himalayan Range which includes the world's third highest peak, Khangchendzonga and down to its south is Singalila ridge. In the north it is bound by Donghia range and also partly includes the Tibetan Plateau. In the east it is bound by the Chola range. The average steepness is about 45 degrees. Sikkim is the main catchment area for the beautiful river Teesta, which has its main source from Chho Lhamo lake in the north and is further strengthened by many streams and rivers of which Tholung, Lachung, Great Rangeet and Rangpo are important drains. It also has about 180 perennial lakes, among which Khecheopalri, Gurudongmar, Chho Lhamo and Men Moi Tso are some of the most scenic.

21. (D) The interests of the Vijayanagar rulers and the Bahamani sultans clashed in three separate and distinct areas: in the Tungabhadra doab, in the Krishna-Godavari delta, and in the Marathwada country. The Tungabhadra doab was the region between the rivers Krishna and Tungabhadra. On account of its wealth and economic resources, it had been the bone of contention between the western Chalukyas and the Cholas in th earlier period and between the Yadavas and the Hoysalas later on.

22. (B) Several tributaries of the Ganga-Yamuna system originate from the Vindhyas. These include Chambal, Betwa, Ken, Kali Sindh and Parbati. The northern slopes of the Vindhyas are drained by these rivers. Most of the tributaries of the river Yamuna have their origin in the Vindhyan and Kaimur ranges. Banas is the only significant tributary of the river Chambal that originates from the Aravalli in the west.

23. (A) When the Lok Sabha is dissolved, all business including bills, motions, resolutions, notices, petitions and so on pending before it or its committees lapse. They (to be pursued further) must be reintroduced in the newly-constituted Lok Sabha. However, some pending bills and all pending assurances that are to be examined by the Committee on Government Assurances do not lapse on the dissolution of the Lok Sabha. The position with respect to lapsing of bills is as follows: A bill pending in the Lok Sabha lapses (whether originating in the Lok Sabha or transmitted to it by the Rajya Sabha). A bill passed by the Lok Sabha but pending in the Rajya Sabha lapses. A bill not passed by the two Houses due to disagreement and if the president has notified the holding of a joint sitting before the dissolution of Lok Sabha, does not lapse. A bill pending in the Rajya Sabha but not passed by the Lok Sabha does not lapse. A bill passed by both Houses but pending assent of the president does not lapse. A bill passed by both Houses but returned by the president for reconsideration of Houses does not lapse.

24. (C) Current Account Deposits are payable on demand. They can be drawn upon by cheque without any restriction. These accounts are usually maintained by businesses and are used for making business payments. No interest is paid on these accounts. Fixed/Term Deposits are deposits for a fixed term (period of time) varying from a few days to a few years. They are not payable on demand and do not enjoy checking facilities. Interest is paid on these deposits and the rate of interest rises with the term of the deposits. In monetary analysis deposits are classified into two types: demand deposits and time deposits. Demand deposits are payable on demand either through cheque or otherwise. Only demand deposits may serve as a medium of exchange, because their ownership can be transferred from person to person through cheques. All other deposits are not payable on demand are called time deposits. All current account deposits are demand deposits and all term deposits are time deposits.

25. (C) While taking a somersault the gymnast travels in a circular path. Because the gymnast is traveling in a circular path, she experiences angular momentum. Angular momentum is the product of angular velocity and the body's moment of inertia (L=Iω). A moment of inertia (I) is how far the mass is from the center of the axis of rotation. Like linear momentum, angular momentum is conserved. By tucking in her knees, the gymnast brings her mass closer to the center of the axis of rotation, thereby decreasing the moment of inertia. When the gymnast decreases her moment of inertia, her angular velocity increases proportionally. Her increased angular velocity allows the gymnast to complete the rotation

26. (D) On the basis of recommendation of the standing committee, the Union Cabinet gave its approval to move official amendments in the Compensatory Afforestation Fund Bill, 2015. Proposed Amendments: It will make the list of environmental services inclusive and will delete some of environmental services for which credible models to assess their monetary value do not exist. It will provide for prior consultation with States Governments for making rule under the new legislation. The Bill establishes the National Compensatory Afforestation Fund under the Public Account of India, and a State Compensatory Afforestation Fund under the Public Account of each state. It will also provide for establishment of State Fund of a Union territory having no legislature under Public Account of the Union of India. It will provide for use of monies realized from the user agencies in lieu for forest land diverted in protected areas for voluntary relocation from protected areas. To provide wider representation, it will include Secretaries of Ministries dealing with Space and Earth Sciences as members of the governing body of the National Authority. It will also increase the number of expert members in the governing body of the National Authority from two to five. It will also increase the number of expert members in the executive committee of National Authority from two to three. It will also include an expert on tribal matters or representative of tribal community as a member in both the steering committee and executive committee of a State Authority. Amendments have a fixed time limit of three months for the Executive Committee of the National Authority to approve the annual plan of operations of State Authorities. These Funds will receive payments for: (i) compensatory afforestation, (ii) net present value of forest (NPV), and (iii) other project specific payments. The National Fund will receive 10% of these funds, and the State Funds will receive the remaining 90%

27. (D) Chimmony (Chimmini) Wildlife Sanctuary is located in the Mukundapuram Taluk of Thrissur district. The Chimmini Wildlife Sanctuary which was established in 1984, lies contiguous with the Peechi - Vazhani sanctuary. Living here in joyful abandon are elephants, sambars, gaurs, Malabar squirrels, sloth bears etc. A unique thorny scrub forest with xerophyte species, Chinnar is the habitat for the endangered Giant Grizzled Squirrel of India. It is located in the rain shadow area of the Western Ghats in Idukki, Kerala. The Chulanur Peafowl (Peacocks) Sanctuary is located in the districts of Thrissur and Palakkad. About 200 peacocks inhabit the extensive forests of the Chulanur Peafowl Sanctuary at Nedungapra near Palakkad.

28. (D) The Peninsular part of India was a part of the Gondwana land. The Gondwanaland included India, Australia, South Africa and South America as one single land mass. The convection currents split the crust into a number of pieces, thus leading to the drifting of the Indo-Australian plate after being separated from the Gondwana land, towards the north.

29. (C) The Safety valve theory suggests that Indian National Congress was started by A.O. Hume under the guidance of Lord Dufferin, to provude a safe, mild, peaceful and constitutional outlet or safety valve for the rising discontent among the masses, which was inevitably leading towards a popular and violent revolution. In his Young India published in 1916, the Extremist leader Lala Lajpat Rai used the safety valve theory to attack the Moderates in the Congress. Having discussed the theory at length and suggested that the Congress was a product of Lord Dufferin's brain,' he argued that the Congress was started more with the object of saving the British empire from danger than with that of wining political liberty of India.

30. (D) Schedule Tribes and Other Forest Dwellers Act or Recognition of Forest Rights Act came into force in 2006. The Nodal Ministry for the Act is Ministry of Tribal Affairs. The Acts seeks to recognize the rights and occupation of traditional forest dwellers and Schedule Tribes in such forests. It provides for diversion of forest land only with the recommendation of the gram sabhas. Also includes right of self-cultivation for livelihood, rights over minor forest produce, community rights such as nistar etc. Reports of alleged violation of the Forest Rights Act by the Odisha Mining Corporation in tribal areas of Odisha have brought the Act under the spotlight.

31. (C) GDP deflator is a measure of inflation. It is the ratio of the value of goods and services an economy produces in a particular year at current prices to that at prices prevailing during any other reference (base) year. It is also called implicit price deflator. Since the deflator covers the entire range of goods and services produced in the economy - as against the limited commodity baskets for the wholesale or consumer price indices or producer price index - it is seen as a more comprehensive measure of inflation.

32. (D) Concentrating Solar Power (CSP) technology utilizes focused sunlight. CSP plants generate electric power by using mirrors to concentrate (focus) the sun's energy and convert it into high-temperature heat. That heat is then channeled through a conventional generator. The plants consist of two parts: one that collects solar energy and converts it to heat, and another that converts the heat energy to electricity. CSP vs PV - technologies Concentrated Solar Thermal systems (CSP), are not the same as Photovoltaic panels; CSP systems concentrate radiation of the sun to heat a liquid substance which is then used to drive a heat engine and drive an electric generator. This indirect method generates alternating current (AC) which can be easily distributed on the power network. Photovoltaic (PV) solar panels differ from solar thermal systems in that they do not use the sun’s heat to generate power. Instead, they use sunlight through the ‘photovoltaic effect’ to generate direct electric current (DC) in a direct electricity production process. The DC is then converted to AC, usually with the use of inverters, in order to be distributed on the power network.

33. (B) The Indian desert lies towards the western margins of the Aravali Hills. It is an undulating sandy plain covered with sand dunes. Barchans, which are crescent shaped dunes, cover larger areas but longitudinal dunes become more prominent near the Indo-Pakistan boundary.

34. (A) The Lilavati is Indian mathematician Bhaskara II's treatise on mathematics, written in 1150. It is the first volume of his main work, the Siddhanta Shiromani, alongside the Bijaganita, the Grahaganita and the Goladhyaya. Sulvasutra prescribes various kinds of measurements for the construction of sacrificial altars. They mark the beginnings of the study of geometry and mathematics. The Charaka Saṃhita or Compendium of Charaka is a text on Ayurveda. Along with the Susruta-saṃhita, it is one of the two foundational Hindu texts of this field that have survived from ancient India

35. (B) Notifying- The state government shall, on recommendation of the National Tiger Conservation Authority, notify an area as a tiger reserve. De-notifying- No state government shall de-notify a tiger reserve, except in public interest with the approval of the National Tiger Conservation Authority and the approval of the National Board of Wildlife.

36. (C) Fiscal Deficit: Fiscal deficit is the difference between the government’s total expenditure and its total receipts excluding borrowing Gross fiscal deficit = Total expenditure - (Revenue receipts + Non-debt creating capital receipts) Primary Deficit: To obtain an estimate of borrowing on account of current expenditures exceeding revenues, we need to calculate what has been called the primary deficit. It is simply the fiscal deficit minus the interest payments Gross primary deficit = Gross fiscal deficit - net interest liabilities Net interest liabilities consist of interest payments minus interest receipts by the government on net domestic lending. Revenue Deficit: The revenue deficit refers to the excess of government’s revenue expenditure over revenue receipts Revenue deficit = Revenue expenditure - Revenue receipts

37. (A) Bonds are instruments of debt that are typically used by corporates to raise money from investors. Masala bonds are rupee-denominated instruments sold only to offshore investors by Indian corporates for raising money from overseas investors. They do not face currency fluctuation risks.

38. (A) The Westminster system is a parliamentary system of government modelled after that which developed in the United Kingdom. From the Westminster system, India has adopted Parliamentary government, Rule of Law, legislative procedure, single citizenship, cabinet system, prerogative writs, parliamentary privileges and bicameralism. US Constitution: Fundamental rights, independence of judiciary, judicial review, impeachment of the president, removal of Sup-reme Court and high court judges and post of vice-president. Canadian Constitution: Federation with a strong Centre, vesting of residuary powers in the Centre, appointment of state governors by the Centre, and advisory jurisdiction of the Supreme Court.

39. (A) The state emblem is an adaptation from the Sarnath Lion Capital of Ashoka. In the original, there are four lions, standing back to back, mounted on an abacus with a frieze carrying sculptures in high relief of an elephant, a galloping horse, a bull and a lion separated by intervening wheels over a bell shaped lotus. Carved out of a single block of polished sandstone, the Capital is crowned by the Wheel of the Law (Dharma Chakra). In the state emblem, adopted by the Government of India on 26 January 1950, only three lions are visible, the fourth being hidden from view. The wheel appears in relief in the centre of the abacus with a bull on right and a horse on left and the outlines of other wheels on extreme right and left. The bell-shaped lotus has been omitted. The words Satyameva Jayate from Mundaka Upanishad, meaning 'Truth Alone Triumphs', are inscribed below the abacus in Devanagari script. Hence A is the correct answer.

40. (C) The rotation period of a celestial object is the time that it takes to complete one revolution around its axis of rotation relative to the background stars. It differs from the planet's solar day, which includes an extra fractional rotation needed to accommodate the portion of the planet's orbital period during one day. A sidereal day represents the time taken by the earth to rotate on its axis relative to the stars, and is almost four minutes shorter than the solar day because of the earth's orbital motion.

41. (D) Dioxins are a group of chemically-related compounds that are persistent environmental pollutants (POPs). Dioxins are found throughout the world in the environment and they accumulate in the food chain, mainly in the fatty tissue of animals. More than 90% of human exposure is through food, mainly meat and dairy products, fish and shellfish. Many national authorities have programmes in place to monitor the food supply. Dioxins are highly toxic and can cause reproductive and developmental problems, damage the immune system, interfere with hormones and also cause cancer. Prevention or reduction of human exposure is best done via source-directed measures, i.e. strict control of industrial processes to reduce formation of dioxins

42. (B) The Nai-Dhobi band was a form of social boycott witnessed during the Kisan Sabha movement in the United Provinces. The landowners and the empathizers of the government were denied the basic facilities of washermen and barber. They were socially boycotted.

43. (A) Muriate of potash is the common name for Potassium Chloride. It is a crystalline solid and dissolves readily in water. The most common use of MOP is as a crop fertilizer, providing the "K" in NPK mix for plant growth. It is also used in medicine, food processing etc. Another commonly used chemical is potassium nitrate (saltpeter) which is used in fertilizers, tree stump removal, rocket propellants, gunpowder and fireworks.

44. (C) The President’s election is held in accordance with the system of proportional representation by means of the single transferable vote and the voting is by secret ballot. This system ensures that the successful candidate is returned by the absolute majority of votes. A candidate, in order to be declared elected to the office of President, must secure a fixed quota of votes. The quota of votes is determined by dividing the total number of valid votes polled by the number of candidates to be elected (here only one candidate is to be elected as President) plus one and adding one to the quotient.

45. (C) The United Nations Environment Programme (UNEP) is the leading global environmental authority that sets the global environmental agenda, promotes the coherent implementation of the environmental dimension of sustainable development within the United Nations system and serves as an authoritative advocate for the global environment. UNEP work encompasses: Assessing global, regional and national environmental conditions and trends Developing international and national environmental instruments Strengthening institutions for the wise management of the environment Global Environment Outlook assessment is the flagship report of UNEP. Recently, the sixth edition of the report made some observations about world climate. It is headquartered at Nairobi, Kenya.

46. (A) A barometer measures air pressure: A "rising" barometer indicates increasing air pressure; a "falling" barometer indicates decreasing air pressure. Sudden fall in the Barometer is a characteristic of stormy weather.

47. (B) Infrastructure Investment Trusts (InvITs) are mutual fund like institutions that enable investments into the infrastructure sector by pooling small sums of money from multitude of individual investors for directly investing in infrastructure so as to return a portion of the income (after deducting expenditures) to unit holders of InvITs, who pooled in the money. InvITs are regulated by the securities market regulator in India- Securities and Exchange Board of India (SEBI). Listing on stock exchange is mandatory for both publicly offered and privately placed InvITs.

Advantages of InvITs o InvITs may help in attracting international finance into Indian infrastructure sector. o InvITs will enable the investors to hold a diversified portfolio of infrastructure assets. o InvITs are also proposed to bring higher standards of governance into infrastructure development and management and distribution of income from assets so as to attract investor interest.

48. (C) Union Ministry of Health and Family Welfare has launched Project Sunrise on prevention of AIDS especially among people injecting drugs in the 8 North-Eastern states. The AIDS prevention special project aims to diagnose 90 per cent of such drug addicts with HIV and put them under treatment by 2020.

49. (B) The Indian merchants were organised in guilds. There were several such guilds in south India from the eighth century onwards – the most famous being the Manigram and Nanadesi. These guilds traded extensively both within the peninsula and with Southeast Asia and China.

50. (D) Xenotransplantation is when living cells, tissues or organs are transplanted between species. Uses of xenotransplantation o Organ transplants - replacing diseased organs, such as hearts, lungs, livers, pancreases and kidneys. o Cell transplants - replacing damaged or destroyed cells in diseases such as diabetes, Alzheimer’s and Parkinson’s disease. o Tissue transplants - skin grafts, cornea transplants or bone transplants. o Bridging transplants - providing organ function externally to patients with organ failure.

51. (C) Directive Principles of State Policy are guidelines or principles to be kept in mind while framing laws and policies .The DPSP are non-justiciable in nature or not enforceable by courts & therefore does not restrict the power of the Parliament. Rest all the other options restricts the power of the Parliament

52. (A) Hill myna is found in Sri Lanka and the Western Ghats of India, east and north-east India east to southern China, and south through south-east Asia to Palawan, Borneo, and Flores. Also native to eastern India, southern China, Indochina, Thailand, Malaysia, and the Philippines. It is extremely noisy, with a vocabulary of whistles, wails and squawks; a superb mimic in captivity. Although the hill mynah is able to mimic the human voice so expertly in captivity, it does not imitate the sounds of other animals when in the wild.

53. (C) Statement 1 is correct. The warm, humid air along the equator rises (because of its low density) and cools, allowing water vapor to condense, which results in rainfall, causing precipitation to exceed evaporation in equatorial regions. Statement 2 is correct. Low salinity water lies in the polar and subpolar regions and near the equator. Toward the poles, fresh water from melting ice decreases the surface salinity.

54. (B) Fiat money is currency that a government has declared to be legal tender, but is not backed by a physical commodity. The value of fiat money is derived from the relationship between supply and demand rather than the value of the material that the money is made of. Historically, most currencies were based on physical commodities such as gold or silver, but fiat money is based solely on faith. Because fiat money is not linked to physical reserves, it risks becoming worthless due to hyperinflation. If people lose faith in a nation's paper currency, like the dollar bill, the money will no longer hold any value. Most modern paper currencies are fiat currencies, have no intrinsic value and are used solely as a means of payment. Historically, governments would mint coins out of a physical commodity such as gold or silver, or would print paper money that could be redeemed for a set amount of physical commodity. Fiat money is inconvertible and cannot be redeemed. Fiat money rose to prominence in the 20th century, specifically after the collapse of the Bretton Woods system in 1971, when the United States ceased to allow the conversion of the dollar into gold.

55. (A) In general, people with diabetes either have a total lack of insulin (type 1 diabetes) or they have too little insulin or cannot use insulin effectively (type 2 diabetes). Type 1 diabetes is an autoimmune disease which is believed to be congenital and affects children and young people unlike Type 2, a late onset disease that afflicts 90 percent of diabetics and is thought to be caused by improper nutrition and other lifestyle factors. India also is in the grip of a Type 2 diabetes epidemic, with more rural than urban populations diagnosed with the condition. There is also a marked rise in Type 1 afflictions.

56. (D) Bardoli Satyagraha of 1928: In 1928, authorities raised the land revenue in Bardoli. The region was already reeling under the grunt of famine and floods. This unjustified levy of tax was opposed by the Congress leaders who set up the Bardoli Inquiry Committee to initiate this movement. Vallabh Bhai Patel was one of the most strong and prominent leaders from Gujarat. As such, he was given the command of this movement. He was respected by the farmers and the entire Gujarati community. Kheda Peasant Struggle (1918): The Kheda peasant struggle is also known as no-tax peasant struggle. It was a satyagraha launched in March 1919 under the leadership of Gandhiji, Sardar Vallabhbhai Patel, Indulal Yajnik, N.M. Joshi, Shankerlal Pareekh and several others. Sardar Vallabhbhai Patel participated in the Nagpur flag satyagraha from May to August in 1923 in protest against the stopping of a procession which carried the national flag. 57. (C) Ecocline (ecological gradient) A gradation from one ecosystem to another when there is no sharp boundary between the two. It is the joint expression of associated community (coenocline) and complex environmental gradients. Ecotone: An ecotone is a transition area between two biomes. It is where two communities meet and integrate. It may be narrow or wide, and it may be local (the zone between a field and forest) or regional (the transition between forest and grassland ecosystems). Ecotype: In evolutionary ecology, an ecotype, sometimes called ecospecies, describes a genetically distinct geographical variety, population or race within a species, which is adapted to specific environmental conditions. They typically show morphological differences. An ecological niche is the role and position a species has in its environment; how it meets its needs for food and shelter, how it survives, and how it reproduces. A species' niche includes all of its interactions with the biotic and abiotic factors of its environment. No two species have identical niches

58. (B) The Central Vigilance Commission (CVC) is the main agency for preventing corruption in the Central government. It was established in 1964 by an executive resolution of the Central government. Its establishment was recommended by the Santhanam Committee on Prevention of Corruption1 (1962-64). Originally the CVC was neither a constitutional body nor a statutory body. Recently, in September 2003, the Parliament enacted a law conferring statutory status on the CVC. The CVC is a multi-member body consisting of a Central Vigilance Commissioner (chairperson) and not more than two vigilance commissioners. They are appointed by the president by warrant under his hand and seal on the recommendation of a three-member committee consisting of the prime minister as its head, the Union minister of home affairs and the Leader of the Opposition in the Lok Sabha. The President can remove the Central Vigilance Commissioner or any vigilance commissioner from the office under the following circumstances: o If he is adjudged an insolvent; or o If he has been convicted of an offence which (in the opinion of the Central government) involves a moral turpitude; or o If he engages, during his term of office, in any paid employment outside the duties of his office; or o If he is (in the opinion of the president), unfit to continue in office by reason of infirmity of mind or body; or o If he has acquired such financial or other interest as is likely to affect prejudicially his official functions. In addition to these, the president can also remove the Central Vigilance Commissioner or any vigilance commissioner on the ground of proved misbehaviour or incapacity. The CVC has to present annually to the President a report on its performance. The President places this report before each House of Parliament.

59. (C) Sangam literature was produced over a period of three to four centuries by poets who assembled in college patronised by chiefs and kings. Such colleges were called Sangam. The Sangam texts are different from the Vedic texts, particularly the Rig Vedic texts. They do not constitute religious literature. The short and long poems were composed by numerous poets in praise of numerous heroes and heroines. They are Secular in nature. The Sangam texts refer to many settlements including Kaveripattinam. They also speak of the Yavanas coming in their own vessels purchasing pepper with gold and supplying wine to the natives.

60. (D) The Advance Pricing Agreement (APA) programme was introduced in India by the Finance Act, 2012. It seeks to provide a predictable and non-adversarial tax regime and to reduce litigation in the Indian transfer pricing regime. The APAs cover a wide range of international transactions, including royalty, corporate guarantees, software development services, IT enabled services and trading. The agreements also pertain to different industrial sectors like media, telecom, automobiles, IT services, etc.

61. (B) The episodic nature of the Earth's glacial and interglacial periods within the present Ice Age (the last couple of million years) have been caused primarily by cyclical changes in the Earth's circumnavigation of the Sun. Variations in the Earth's eccentricity, axial tilt, and precession comprise the three dominant cycles, collectively known as the Milankovitch Cycles. It is of primary importance to explain that climate change, and subsequent periods of glaciation, resulting from the following three variables is not due to the total amount of solar energy reaching Earth. The three Milankovitch Cycles impact the seasonality and location of solar energy around the Earth, thus impacting contrasts between the seasons.

62. (C) In a vacuum container air friction will be absent. Hence gravitational force alone will determine the time of vertical drop. The acceleration ‘g’, initial speed and vertical height to be covered, all are same for both the articles. According to kinematics equations: Distance = Initial speed*t + ½*g*t2 t = time of flight; g = gravitational acceleration Thus “t” would be the same for both the articles.

63. (C) Although Uttarakhand has seen a spike in forest fires in both high-count years, 2012 and 2016, there are 12 states that have registered more forest fires than it in the last five years. Large parts of the northeast (where the practice of shifting cultivation is common) and Odisha are affected every year, as are parts of Central India. In the last five years, the states to have registered the maximum number of forest incidents are Odisha (10,636), Mizoram (10,335), Assam (9,602), Chhattisgarh (9,210) and Maharashtra (7,534).

In the last five years, 2012 saw the maximum incidents of forest fires, though 2016 is running close. Typically, forest fires peak in March: in the last five years, the month accounted for between 46% and 66% of forest fire incidents. Wildfires are sometimes a natural process, and help forests by promoting flowering, branching and seedling establishment. Himachal Principal Chief Conservator of Forests S P Vasudeva says fires that are limited to the surface may help in the natural regeneration of forests. The heating of the soil may result in helpful microbial activity, and hasten decaying processes that are useful for the vegetation.

64. (A) The United Nations Economic and Social Commission for Asia and the Pacific (ESCAP) is the regional development arm of the United Nations for the Asia-Pacific region. Made up of 53 Member States and 9 Associate Members, with a geographical scope that stretches from Turkey in the west to the Pacific island nation of Kiribati in the east, and from the Russian Federation in the north to New Zealand in the south, the region is home to 4.1 billion people, or two thirds of the world’s population.

65. (C) Like the Speaker, the Deputy Speaker is also elected by the assembly itself from amongst its members. He is elected after the election of the Speaker has taken place. Like the Speaker, the Deputy Speaker remains in office usually during the life of the assembly. However, he also vacates his office earlier in any of the following three cases: o if he ceases to be a member of the assembly; o if he resigns by writing to the speaker; and o if he is removed by a resolution passed by a majority of all the then members of the assembly. Such a resolution can be moved only after giving 14 days’ advance notice. The Deputy Speaker performs the duties of the Speaker’s office when it is vacant. He also acts as the Speaker when the latter is absent from the sitting of assembly. In both the cases, he has all the powers of the Speaker.

66. (C) ‘Inflation’ refers to a sustained, across-the-board price increase, whereas ‘a relative price increase’ is a reference to an episodic price rise pertaining to one or a small group of commodities. This leaves a third phenomenon, namely one in which there is a price rise of one or a small group of commodities over a sustained period of time, without a traditional designation. ‘Skewflation’ is a relatively new term to describe this third category of price rise. The distinction between these different kinds of inflation is important because they call for different kinds of policy response from the government.

67. (A) Classification of Plateau: Intermontane Plateau: These Plateaus are surrounded by hills and mountains from all sides. For Example Tibetan plateau, Columbian plateau. Piedmont Plateau: Which is surrounded by-mountain range on one side and by plain or ocean on the other side. For Example Appalachian Mountains (USA) and Patagonian plateau of South America. Dome Shaped Plateau: These Plateaus are formed when the landmass is uplifted in such a manner that the middle portion is raised and the sides are rounded. Chota Nagpur plateau of Jharkhand, Ozark Plateau (USA) Continental Plateau: These are very extensive Plateau and surrounded by oceanic coasts or plains. These are also called Shield. For Example Siberian shield. Volcanic Plateau: These Plates are formed due to accumulation of thick layers of basaltic lavas. The Deccan plateau of India and the Columbian plateau of the USA are the best examples of this type.

68. (D) Pabna Movement (1872-76): In East Bengal the peasantry was oppressed by zamindars through frequent recourse to ejection, harassment, arbitrary enhancement of rent through ceases (abwabs) and use of force. The zamindars also tried to prevent them from acquiring the occupancy rights under the Act of 1859. In May 1873 an Agrarian League was formed in the Yusufzai Pargana of Pabna district (East Bengal). Payments of enhanced rents were refused and the peasants fought the zamindars in the courts. The main form of the struggle was that of legal resistance. Similar leagues were formed in the adjoining districts of Bengal. The main leaders of the Agrarian League were Ishan Chandra Roy, Shambu Pal and Khoodi Mullah. The agrarian leagues kept within the bounds of law, used the legal machinery to fight the zamindars and raised no anti- British demands. The leaders often argued that they leaders often argued that they were against zamindars and not the British. Infact, the leaders raised the slogan that the peasants want to be the ryots of Her Majesty and the Queen and of Her only. The discontent continued till 1885 when the Government by the Bengal Tenancy Act of 1885 enhanced the occupancy rights.

69. (C) Pattachitra is a traditional painting style that originated and flourished in Odisha. The painting depicts mythological characters on treated cloth. Patua art belongs to Bengal and goes back 1000 years. The typical paintings of Sikkim are the thangkas, originally a medium of reverence through which the highest ideals of Buddhism were evoked. Kohvar and Sohrai paintings from Jharkhand are delicate and beautiful.

70. (C) Gulf of Mannar Marine National Park consists of 21 small islands and coral reefs in the Gulf of Mannar in the Indian Ocean of Tamil Nadu. The park has a high diversity of plants and animals in its marine and shore habitats, Dugong, a vulnerable marine mammal also found here. Gahirmatha is the first and the only Marine Sanctuary of Orissa, a number of marvelous creatures including Olive Ridley sea turtles migrate in huge numbers. Gahirmatha Beach separates the Bhitarkanika mangroves from the Bay of Bengal is the world’s most important nesting beach for Olive Ridley Sea Turtles. Mahatma Gandhi Marine National Park is a national park of India on the Andaman Islands, Situated 29 km. from Port Blair. Most of the coral reefs in the park are fringing reefs and the park is an important breeding ground for turtles. Therefore, it is not located off the coast of mainland India. Malvan Marine Sanctuary is the only marine sanctuary located in Malvan Taluka of Sindhudurg district in Konkan region of Maharashtra. The Malvan Wildlife Sanctuary includes Padang island and other submerged rocky structures.

71. (C) Radio frequency identification, or RFID, is a generic term for technologies that use radio waves to automatically identify people or objects. There are several methods of identification, but the most common is to store a serial number that identifies a person or object, and perhaps other information, on a microchip that is attached to an antenna (the chip and the antenna together are called an RFID transponder or an RFID tag). The antenna enables the chip to transmit the identification information to a reader. The reader converts the radio waves reflected back from the RFID tag into digital information that can then be passed on to computers that can make use of it. RFID readers do not require a direct line of sight. RFID tags can be read at much greater distances; an RFID reader can pull information from a tag at distances up to 300 feet. RFID readers can interrogate, or read, RFID tags much faster; read rates of forty or more tags per second are possible. RFID tags can be read/write devices; the RFID reader can communicate with the tag, and alter as much of the information as the tag design will allow.

72. (D) Except Article 15, 16, 19, 29 and 30 all other fundamental rights are available to both citizens and foreigners. Thus a foreigner doesn't enjoy- Equality of opportunity in matters of public employment (Article 16), Protection of six rights regarding freedom of: o speech and expression, o assembly, o association, o movement, o residence, and o profession (Article 19), Right of minorities to establish and administer educational institutions (Article 30). Thus he can't claim Fundamental Right for any of the given issues.

73. (D) A Collective Investment Scheme (CIS), as its name suggests, is an investment scheme wherein several individuals come together to pool their money for investing in a particular asset(s) and for sharing the returns arising from that investment as per the agreement reached between them prior to pooling in the money. The term has broader connotations and includes even mutual funds. For instance, in the UK, the unit trust scheme is a collective investment scheme. However, in India, as in US, the definition of CIS excludes mutual funds or unit trust schemes etc and is given a strict definition in Section 11AA of the SEBI Act, 1992. CISs are regulated by the securities market regulator - SEBI - under SEBI (Collective Investment Scheme) Regulations, 1999. The following do not constitute a collective investment scheme: o any scheme or arrangement made or offered by a co-operative society or a society being a society registered or deemed to be registered under any law relating to co-operative societies for the time being in force in any State. o any scheme or arrangement under which deposits are accepted by non-banking financial companies o any scheme or arrangement being a contract of insurance to which the Insurance Act, applies o any scheme or arrangement falling within the meaning of Chit business as defined in clause (d) of section 2of the Chit Fund Act, 1982 (40 of 1982)

74. (A) The correct sequence from West to east is: Mizo Hills, Barail Range, Naga Hills, Mishmi Hills. Some details about them are: Mizo Hills: Mizo Hills, formerly Lushai Hills, mountain range in southeastern Mizoram state, northeastern India, forming part of the north Arakan Yoma system. The Mizo Hills rise to about 7,000 feet (2,125 metres), and their slopes are covered with thick evergreen forest containing valuable timber and bamboo. In the intermontane valleys, shifting (slash-and-burn) agriculture and some terrace cultivation are practiced. Barail range: It lies on the Assam Nagaland border. The Barak river originates from this range. Naga Hills: Naga Hills, part of the complex mountain barrier on the border of India and Myanmar (Burma). A northern extension of the Arakan Yoma system, the Naga Hills reach a height of 12,552 feet (3,826 m) in Mount Saramati on the India-Myanmar frontier. Mishmi Hills: Mishmi Hills are on the Northeastern tip of India, in central Arunachal Pradesh. The hills are a southward extension of the Great Himalayan ranges and its northern and eastern parts touch China.

75. (C) Integrated Gasification Combined Cycle IGCC is a combination of two leading technologies. The first technology is called coal gasification, which uses coal to create a clean-burning gas (syngas). The second technology is called combined-cycle, which is the most efficient method of producing electricity commercially available today. Coal Gasification: The gasification portion of the IGCC plant produces a clean coal gas (syngas) which fuels the combustion turbine. After cleaning, the coal gas is used in the combustion turbine to produce electricity. Combined-cycle: This design consists of a combustion turbine/generator, a heat recovery steam generator, and a steam turbine/generator. The exhaust heat from the combustion turbine is recovered in the heat recovery steam generator to produce steam. This steam then passes through a steam turbine to power another generator, which produces more electricity. Combined cycle is more efficient than conventional power generating systems because it re-uses waste heat to produce more electricity. The integration of these technologies provides the high efficiency of the combined cycle design with the low cost of coal for fuel.

76. (B) The Union Ministry of Health and Family Welfare launched India’s first indigenous rotavirus vaccine named Rotavac to combat infant mortality due to diarrhea. The Rotavirus vaccine has been developed indigenously under a public-private partnership between the Union Ministry of Science Technology, Union Health Ministry, institutions of the US Government and NGOs in India supported by the Bill and Melinda Gates Foundation. The vaccine is administered orally to infants in three dose course at ages of six, ten and fourteen weeks as part of Universal Immunisation Programme (UIP). Diarrhea is caused by a virus which spreads from person to person due to bacterial and parasitic agents that are primarily transmitted through contaminated food or water. It causes gastroenteritis after it damages the cells that line the small intestine and causes gastroenteritis. In some cases it causes malnutrition, delayed physical and mental development among children.

77. (B) Ajivika is one of the nastika or "heterodox" schools of Indian philosophy. Founded in the 5th century BCE by Makkhali Gosala, it was a śramaṇa movement and a major rival of early Buddhism and . Ajivika were organised renunciates who formed discrete communities. Original scriptures of the Ajivika school of philosophy once existed, but these are unavailable and probably lost. Their theories are extracted from mentions of Ajivikas in the secondary sources of ancient Indian literature. The Ajivika school is known for its Niyati doctrine of absolute determinism, the premise that there is no free will, that everything that has happened, is happening and will happen is entirely preordained and a function of cosmic principles. Ajivika considered the karma doctrine as a fallacy.

78. (C) LVM 3 is a heavy launch capability launcher being developed by ISRO. It will allow India to achieve complete self reliance in launching satellites as it will be capable of placing 4 tonne class Geosynchronous satellites into orbit. The LVM3 will have an India built cryogenic stage with higher capacity than GSLV. The powerful cryogenic stage of LVM3 enables it to place heavy payloads into Low Earth Orbits of 600 km altitude. The first experimental flight of LVM3, the LVM3-X/CARE mission lifted off from Sriharikota in December, 2014 and successfully tested the atmospheric phase of flight. The core stage of the launch is the L110 liquid stage which is powered by two Vikas engines designed and developed at the Liquid Propulsion Systems Centre. Crew module Atmospheric Reentry Experiment was also carried out in this flight. The module reentered, deployed its parachutes as planned and splashed down in the Bay of Bengal.

79. (A) The rough side slows down the air due to friction whereas the smoother side allows the air to pass by at a greater speed. The greater the air speed, the less the pressure. So there is a pressure differential that pushes the ball in the direction of the smoother side

80. (A) Union Government has launched Setu Bharatam Project to make all national highways free of railway crossings by 2019. This project will be implemented by the Union Ministry of Road Transport and Highways. Under this project 208 places have been identified for construction of rail overbridges (ROB) or underpasses under the project. Around 1,500 bridges that are over 50 to 60 years old or of the British era will be overhauled and will be rebuilt.

81. (C) Article 356 empowers the president to issue a proclamation if he is satisfied that a situation has arisen in which the government of a state cannot be carried on in accordance with the provisions of the constitution. Notably, the president can either act on a report of the governor of a state or otherwise too (even without governor’s report). A proclamation of president's rule may be revoked by the president at any time by a subsequent proclamation. Such a proclamation doesn't require the parliamentary approval. A law made by Parliament or president or any other specified authority continues to be operative even after the president’s rule.

82. (D) Central Road Fund (CRF) is a non-lapsable fund created under Section 6 of the Central Road Fund Act, 2000 out of a cess/tax imposed by the Union Government on the consumption of Petrol and High Speed Diesel to develop and maintain National Highways, State roads (particularly those of economic importance and which provides inter-state connectivity), rural roads, railway under/over bridges etc.

83. (B) The Treaty on the Non-Proliferation of Nuclear Weapons, also referred to as the Nuclear NonProliferation Treaty (NPT), obligates the five acknowledged nuclear-weapon states (the United States, Russian Federation, United Kingdom, France, and China) not to transfer nuclear weapons, other nuclear explosive devices, or their technology to any non-nuclear-weapon state. The NPT is the most widely accepted arms control agreement; only Israel, India, and Pakistan have never been signatories of the Treaty, and North Korea withdrew from the Treaty in 2003. South Sudan is also not a member of this treaty.

84. (D) Each water use has specific quality needs. Therefore, to set the standard for the desire quality of a water body, it is essential to identify the uses of water in that water body. In India, the Central Pollution Control Board (CPCB) has developed a concept of designated best use. According to this, out of the several uses of water of a particular body, the use which demands highest quality is termed its designated best use. Five designated best uses have been identified. This classification helps the water quality managers and planners to set water quality targets and design suitable restoration programs for various water bodies. Several parameters set by CPCB to determine designated best use . For example for drinking water the parameters set are: Total Coliform content, Acidity (pH),Amount of dissolved oxygen and Biochemical Oxygen Demand.

85. (A) LCD stands for “liquid crystal display” and technically, both LED and LCD TVs are liquid crystal displays. The basic technology is the same in that both television types have two layers of polarized glass through which the liquid crystals both block and pass light. So really, LED TVs are a subset of LCD TVs. LED, which stands for “light emitting diodes,” differs from general LCD TVs in that LCDs use fluorescent lights while LEDs use those light emitting diodes. Also, the placement of the lights on an LED TV can differ. The fluorescent lights in an LCD TV are always behind the screen. On an LED TV, the light emitting diodes can be placed either behind the screen or around its edges. The difference in lights and in lighting placement has generally meant that LED TVs can be thinner than LCDs, although this is starting to change. It has also meant that LED TVs run with greater energy efficiency and can provide a clearer, better picture than the general LCD TVs.

86. (B) Indian Social Conference was founded by M.G. Ranade and Raghunath Rao. It focused attention on the social issues of importance. The conference advocated inter-caste marriages, opposed polygamy and kulinism. It launched the "Pledge Movement" to inspire people to take a pledge against child marriage. However it did not aim to bring together all the social reform groups of the country under one organisation.

87. (B) Africa and Eurasia touches at three point i.e. Strait of Gibraltar, Suez Canal and Bab-El-Mandeb. The Strait of Hormuz is a strait between the Gulf of Oman and the Persian Gulf. It provides the only sea passage from the Persian Gulf to the open ocean and is one of the world's most strategically important choke points.

88. (C) Committee on Absence of Members: It considers all applications from members for leave of absence from the sittings of the House; and examines the cases of members who had been absent for a period of 60 days or more without permission. It is a special committee of the Lok Sabha and consists of 15 members. There is no such committee in the Rajya Sabha and all such matters are dealt by the House itself. Committee of Privileges: Its functions are semi-judicial in nature. It examines the cases of breach of privileges of the House and its members and recommends appropriate action. The Lok Sabha committee has 15 members, while the Rajya Sabha committee has 10 members. Committee on Private Members’ Bills and Resolutions: This committee classifies bills and allocates time for the discussion on bills and resolutions introduced by private members (other than ministers). This is a special committee of the Lok Sabha and consists of 15 members including the Deputy Speaker as its chairman. The Rajya Sabha does not have any such committee. The same function in the Rajya Sabha is performed by the Business Advisory Committee of that House.

89. (A) Open Acreage Licensing Policy (OALP) gives an option to a company looking for exploring hydrocarbons to select the exploration blocks on its own, without waiting for the formal bid round from the Government. Under Open Acreage Licensing Policy (OALP), a bidder intending to explore hydrocarbons like oil and gas, coal bed methane, gas hydrate etc., may apply to the Government seeking exploration of any new block (not already covered by exploration). The Government will examine the Expression of Interest and justification. If it is suitable for award, Govt. will call for competitive bids after obtaining necessary environmental and other clearances.

90. (A) Llanos is a vast tropical grassland plain situated to the east of the Andes in northwestern South America. They are wide grasslands stretching across northern South America and occupying western Venezuela and northeastern Colombia. The Pampas of South America are a grassland biome. They are flat, fertile plains that are found primarily in Argentina and extends into Uruguay. Campos is a stretch of tropical grasslands in Brazil.

91. (D) Kalighat Painting: In the nineteenth century a new world of popular art developed in many of the cities of India. In Bengal, around the pilgrimage centre of the temple of Kalighat, local village scroll painters (called patuas) and potters (called kumors in eastern India and kumhars in north India) began developing a new style of art. Before the nineteenth century, the village patuas and kumors had worked on mythological themes and produced images of gods and goddesses. After the 1840s, we see a new trend within the Kalighat artists. Living in a society where values, tastes, social norms and customs were undergoing rapid changes, Kalighat artists responded to the world around, and produced paintings on social and political themes. Many of the late-nineteenth century Kalighat paintings depict social life under British rule. Often the artists mocked at the changes they saw around, ridiculing the new tastes of those who spoke in English and adopted Western habits, dressed like sahibs, smoked cigarettes, or sat on chairs. Kalighat painters began to use shading to give them a rounded form, to make the images look three dimensional.

92. (B) The quality of air in and around buildings is seriously affected by gases (like CO2, CO, radon, volatile organic compounds), particulates, microbial contaminants or any mass or energy stressor that can induce adverse health conditions. In recent years the health problems due to indoor pollution is increasing, a syndrome called Sick Building Syndrome (SBS). Source control, filtration and the use of ventilation to dilute contaminants are the primary methods for improving indoor air quality in most buildings.

93. (A) The important tributaries of the Yamuna River are Tons, Chambal, Hindon, Betwa and Ken. Other small tributaries of the Yamuna River include the Giri, Sind, Uttangan, Sengar and the Rind. The main Yamuna and Tons are fed by glaciers, viz., the BandarPunch Glacier and its branches and originates from the Great Himalayan range. Son River of central India is the Second largest of the Ganges' southern tributaries after Yamuna River.

94. (B) Relative velocity remains the same between the moving bus/train and the objects. However it is the angular velocity of the objects with respect to the moving vehicle that varies with distance. It decreases with increasing distance from the moving objects. That's why objects at distance appear moving slow as compared to nearer objects. Here one can see that angular velocity is inversely proportional to distance(r).

95. (A) A person is registered as Overseas Citizen of India (OCI) under the Citizenship Act, 1955. Following benefits are available to OCIs: A multiple entry, multi-purpose life long visa for visiting India. Exemption from registration with local police authority for any length of stay in India. Parity with NRIs in respect of economic, financial and educational fields except in relation to the acquisition of agricultural or plantation properties.

96. (B) The government has given its nod to the ‘Comprehensive Integrated Border Management System’ (CIBMS) for 24x7x365 surveillance of the border(western border with Pakistan) through technology. It is a five-layer elaborate plan to completely stop infiltration on the 2,900-km western border with Pakistan. Five layers include: CCTV cameras. Thermal image and night-vision devices. Battlefield surveillance radar. Underground monitoring sensors. Laser barriers.

97. (B) The border treaty signed between the then government of East India Company and Nepal on March 4, 1816 is known as the Sugauli Treaty (The treaty will complete its 200 years of signing in 2016). There are both direct and indirect consequences of the treaty on Nepal. Direct consequences: The treaty reduced the average east-west length to 885 kilometres and the total area of Nepal is confined to 147,181 square kilometers between the Mechi and the Mahakali Rivers. This way the treaty cut-off the wings on the east and west and receded its area on the south, losing almost one-third of its total area. Nepal was forced to give up not only its western front but also the Mechi to Teesta area (Tarai area)on the east, where there was no war. What was more spiteful of the British was that it entered into Titliyan Treaty with Sikkim on February 10, 1817 (11 months after Sugauli Treaty), and gave the land it had snatched away from Nepal to Sikkim. Nepal also lost the right to deploy any American or European employee in its service. The British representative in Kathmandu was to be posted after the treaty. Treaty of Salbai, 1782- was a treaty to end the first Anglo- Maratha war signed between British and Marathas.

Treaty of Allahabad, 1765- was signed between Robert Clive and Shah Alam II allowing British the Diwan rights in Bengal, Bihar and Orissa. Treaty of Mangalore, 1784- was signed to end the second Anglo Mysore war.

98. (D) Typically, higher inflation is caused by strong economic growth. If Aggregate Demand in an economy expanded faster than aggregate supply, we would expect to see a higher inflation rate. If demand is rising faster than supply this suggests that economic growth is higher than the long run sustainable rate of growth. With high growth, demand rises faster than firms can keep pace with supply; faced with supply constraints, firms push up prices. High growth leads to more employment. Unemployment falls, but this may cause labour shortages. This fall in unemployment puts upward pressure on wages which leads to higher inflation. Higher growth also leads to increasing wages leading to more aggregate demand due to increase in disposable income. This also contributes to inflation.

99. (C) While Bats are mammals, they belong to Phylum Chordata. All other three belong to Phylum Arthropoda.

100. (B) Manipur is a melting pot of diverse cultures and ethnicities and has gradually evolved into a dynamic multicultural society. Naga, Kuki and Meitei stayed together side by side for centuries and developed their heritage in parallel within their respective ethno-social boundaries.

Watch Daily Videos For Answer Writing By BA NAGESH ( Retd Additional Secretary ) Unique Points Shared From His Administrative Experience To Score High Marks in Mains Exam: https://www.youtube.com/c/IASNETWORK

PROGRAM COVERS :

Overall Guidance / Mentorship

Prelims Test Series ( 20+ Full Syllabus Test )

Prelims & Mains Monthly CA

GS Answer and Essay Evaluation ( Questions and Video Explanations Being Provided, Students Can Use Any Other Source Of Their Choice Also )

Fee : Rs 5999 ( One time payment, Valid till Mains 2021 )

Contact : 9779726117

Program Covers :

GS Answer and Essay Evaluation ( Questions and Video Explanations Being Provided, Students Can Use Any Other Source Of Their Choice Also )

Rs 1000 / Month ( 2 Ans / Day + 1 Essay / Week )

Rs 6000 ( No Strict Limits on Answer and Essay, One time payment valid till Mains 2020 )

Contact : 9779726117